Calculate the factor increase for y=ln x












1












$begingroup$


Question:
Suppose $x >1$ and $x$ increases by a factor of $5$. By what factor will $y$ increase given $y = ln(x)$?



My answer:



So the increase would be $$frac{(ln(5x) - ln x)}{ln x} times 100%$$



And then $$frac{( ln 5 + ln x -ln x )}{ln x}times 100 %$$



But thats clearly wrong because $ln x$ get cancelled in the numerator but the denominator is $ln x$










share|cite|improve this question











$endgroup$












  • $begingroup$
    Why must that be wrong? The factor of increase in $y$ will not be independent of $x$. Try some numerical examples. (Please edit the question to reformat with mathjax: math.meta.stackexchange.com/questions/5020/…
    $endgroup$
    – Ethan Bolker
    Dec 13 '18 at 17:26


















1












$begingroup$


Question:
Suppose $x >1$ and $x$ increases by a factor of $5$. By what factor will $y$ increase given $y = ln(x)$?



My answer:



So the increase would be $$frac{(ln(5x) - ln x)}{ln x} times 100%$$



And then $$frac{( ln 5 + ln x -ln x )}{ln x}times 100 %$$



But thats clearly wrong because $ln x$ get cancelled in the numerator but the denominator is $ln x$










share|cite|improve this question











$endgroup$












  • $begingroup$
    Why must that be wrong? The factor of increase in $y$ will not be independent of $x$. Try some numerical examples. (Please edit the question to reformat with mathjax: math.meta.stackexchange.com/questions/5020/…
    $endgroup$
    – Ethan Bolker
    Dec 13 '18 at 17:26
















1












1








1





$begingroup$


Question:
Suppose $x >1$ and $x$ increases by a factor of $5$. By what factor will $y$ increase given $y = ln(x)$?



My answer:



So the increase would be $$frac{(ln(5x) - ln x)}{ln x} times 100%$$



And then $$frac{( ln 5 + ln x -ln x )}{ln x}times 100 %$$



But thats clearly wrong because $ln x$ get cancelled in the numerator but the denominator is $ln x$










share|cite|improve this question











$endgroup$




Question:
Suppose $x >1$ and $x$ increases by a factor of $5$. By what factor will $y$ increase given $y = ln(x)$?



My answer:



So the increase would be $$frac{(ln(5x) - ln x)}{ln x} times 100%$$



And then $$frac{( ln 5 + ln x -ln x )}{ln x}times 100 %$$



But thats clearly wrong because $ln x$ get cancelled in the numerator but the denominator is $ln x$







logarithms






share|cite|improve this question















share|cite|improve this question













share|cite|improve this question




share|cite|improve this question








edited Dec 13 '18 at 19:33









Emilio Novati

52.1k43474




52.1k43474










asked Dec 13 '18 at 17:22









LJacobLJacob

61




61












  • $begingroup$
    Why must that be wrong? The factor of increase in $y$ will not be independent of $x$. Try some numerical examples. (Please edit the question to reformat with mathjax: math.meta.stackexchange.com/questions/5020/…
    $endgroup$
    – Ethan Bolker
    Dec 13 '18 at 17:26




















  • $begingroup$
    Why must that be wrong? The factor of increase in $y$ will not be independent of $x$. Try some numerical examples. (Please edit the question to reformat with mathjax: math.meta.stackexchange.com/questions/5020/…
    $endgroup$
    – Ethan Bolker
    Dec 13 '18 at 17:26


















$begingroup$
Why must that be wrong? The factor of increase in $y$ will not be independent of $x$. Try some numerical examples. (Please edit the question to reformat with mathjax: math.meta.stackexchange.com/questions/5020/…
$endgroup$
– Ethan Bolker
Dec 13 '18 at 17:26






$begingroup$
Why must that be wrong? The factor of increase in $y$ will not be independent of $x$. Try some numerical examples. (Please edit the question to reformat with mathjax: math.meta.stackexchange.com/questions/5020/…
$endgroup$
– Ethan Bolker
Dec 13 '18 at 17:26












1 Answer
1






active

oldest

votes


















1












$begingroup$

The increase in $y$, is just given by $ Delta y=ln(5x)-ln x=ln 5$, which is independent of the value of $x$.



The relative increase in $y$ is given by $frac{Delta y}y=ln5/ln x$, which is not independent of $x$, and can also be expressed as a percentage.






share|cite|improve this answer









$endgroup$













    Your Answer





    StackExchange.ifUsing("editor", function () {
    return StackExchange.using("mathjaxEditing", function () {
    StackExchange.MarkdownEditor.creationCallbacks.add(function (editor, postfix) {
    StackExchange.mathjaxEditing.prepareWmdForMathJax(editor, postfix, [["$", "$"], ["\\(","\\)"]]);
    });
    });
    }, "mathjax-editing");

    StackExchange.ready(function() {
    var channelOptions = {
    tags: "".split(" "),
    id: "69"
    };
    initTagRenderer("".split(" "), "".split(" "), channelOptions);

    StackExchange.using("externalEditor", function() {
    // Have to fire editor after snippets, if snippets enabled
    if (StackExchange.settings.snippets.snippetsEnabled) {
    StackExchange.using("snippets", function() {
    createEditor();
    });
    }
    else {
    createEditor();
    }
    });

    function createEditor() {
    StackExchange.prepareEditor({
    heartbeatType: 'answer',
    autoActivateHeartbeat: false,
    convertImagesToLinks: true,
    noModals: true,
    showLowRepImageUploadWarning: true,
    reputationToPostImages: 10,
    bindNavPrevention: true,
    postfix: "",
    imageUploader: {
    brandingHtml: "Powered by u003ca class="icon-imgur-white" href="https://imgur.com/"u003eu003c/au003e",
    contentPolicyHtml: "User contributions licensed under u003ca href="https://creativecommons.org/licenses/by-sa/3.0/"u003ecc by-sa 3.0 with attribution requiredu003c/au003e u003ca href="https://stackoverflow.com/legal/content-policy"u003e(content policy)u003c/au003e",
    allowUrls: true
    },
    noCode: true, onDemand: true,
    discardSelector: ".discard-answer"
    ,immediatelyShowMarkdownHelp:true
    });


    }
    });














    draft saved

    draft discarded


















    StackExchange.ready(
    function () {
    StackExchange.openid.initPostLogin('.new-post-login', 'https%3a%2f%2fmath.stackexchange.com%2fquestions%2f3038331%2fcalculate-the-factor-increase-for-y-ln-x%23new-answer', 'question_page');
    }
    );

    Post as a guest















    Required, but never shown

























    1 Answer
    1






    active

    oldest

    votes








    1 Answer
    1






    active

    oldest

    votes









    active

    oldest

    votes






    active

    oldest

    votes









    1












    $begingroup$

    The increase in $y$, is just given by $ Delta y=ln(5x)-ln x=ln 5$, which is independent of the value of $x$.



    The relative increase in $y$ is given by $frac{Delta y}y=ln5/ln x$, which is not independent of $x$, and can also be expressed as a percentage.






    share|cite|improve this answer









    $endgroup$


















      1












      $begingroup$

      The increase in $y$, is just given by $ Delta y=ln(5x)-ln x=ln 5$, which is independent of the value of $x$.



      The relative increase in $y$ is given by $frac{Delta y}y=ln5/ln x$, which is not independent of $x$, and can also be expressed as a percentage.






      share|cite|improve this answer









      $endgroup$
















        1












        1








        1





        $begingroup$

        The increase in $y$, is just given by $ Delta y=ln(5x)-ln x=ln 5$, which is independent of the value of $x$.



        The relative increase in $y$ is given by $frac{Delta y}y=ln5/ln x$, which is not independent of $x$, and can also be expressed as a percentage.






        share|cite|improve this answer









        $endgroup$



        The increase in $y$, is just given by $ Delta y=ln(5x)-ln x=ln 5$, which is independent of the value of $x$.



        The relative increase in $y$ is given by $frac{Delta y}y=ln5/ln x$, which is not independent of $x$, and can also be expressed as a percentage.







        share|cite|improve this answer












        share|cite|improve this answer



        share|cite|improve this answer










        answered Dec 13 '18 at 17:31









        Shubham JohriShubham Johri

        5,204718




        5,204718






























            draft saved

            draft discarded




















































            Thanks for contributing an answer to Mathematics Stack Exchange!


            • Please be sure to answer the question. Provide details and share your research!

            But avoid



            • Asking for help, clarification, or responding to other answers.

            • Making statements based on opinion; back them up with references or personal experience.


            Use MathJax to format equations. MathJax reference.


            To learn more, see our tips on writing great answers.




            draft saved


            draft discarded














            StackExchange.ready(
            function () {
            StackExchange.openid.initPostLogin('.new-post-login', 'https%3a%2f%2fmath.stackexchange.com%2fquestions%2f3038331%2fcalculate-the-factor-increase-for-y-ln-x%23new-answer', 'question_page');
            }
            );

            Post as a guest















            Required, but never shown





















































            Required, but never shown














            Required, but never shown












            Required, but never shown







            Required, but never shown

































            Required, but never shown














            Required, but never shown












            Required, but never shown







            Required, but never shown







            Popular posts from this blog

            Plaza Victoria

            In PowerPoint, is there a keyboard shortcut for bulleted / numbered list?

            How to put 3 figures in Latex with 2 figures side by side and 1 below these side by side images but in...